LSAT and Law School Admissions Forum

Get expert LSAT preparation and law school admissions advice from PowerScore Test Preparation.

 Administrator
PowerScore Staff
  • PowerScore Staff
  • Posts: 8917
  • Joined: Feb 02, 2011
|
#73942
Complete Question Explanation

Weaken—CE. The correct answer choice is (B)

The premises contain correlations, and the conclusion makes a causal claim:

..... ..... PC = adequate prenatal care
..... ..... DR = decrease risk of low birth weight babies

..... ..... ..... ..... C ..... E
..... ..... ..... ..... PC :arrow: DR

The question stem asks you to weaken the argument, and the correct answer falls into one of the five
basic methods for weakening a causal argument.

Answer choice (A): The conclusion specifically states that mothers who had received adequate
prenatal care were less likely to have low birth weight babies than mothers who had received
inadequate prenatal care. Thus, although mothers who received inadequate prenatal care have a
higher likelihood of having low birth weight babies, this likelihood still allows for many babies to
be born of normal weight. In a later chapter we will explore the ways the LSAT uses numbers and
statistics to confuse test takers, but for now, consider this analogy: The Detroit Tigers are more likely
to lose a baseball game than any other team, but even so, they can still win a number of games. In
the same way, the aforementioned mothers may be more likely to have low birth weight babies, but
they can still give birth to babies of normal weight. Hence, answer choice (A) does not attack the
argument.

Answer choice (B): This is the correct answer. The answer choice falls into the category of “Showing
a statistical problem exists with the data used to make the causal statement.” By indicating that
mothers without prenatal care records are routinely classified as mothers receiving inadequate
prenatal care, the answer undermines the relationship in the argument because the data used to make
the conclusion is unreliable.

Answer choice (C): The conclusion is about low birth weight babies, not premature babies. Even if
low birth weight babies were routinely classified as premature, that would not affect the conclusion.

Answer choice (D): Similar to answer choice (A), the likelihoods discussed in the stimulus allow for
this possibility. Hence, this answer cannot hurt the argument.

Answer choice (E): If anything, this answer strengthens the argument since it shows that adequate
prenatal care has a powerful positive effect.
 cboles
  • Posts: 27
  • Joined: Sep 15, 2016
|
#28786
Can someone explain why B is the right answer? I chose D
 Nikki Siclunov
PowerScore Staff
  • PowerScore Staff
  • Posts: 1362
  • Joined: Aug 02, 2011
|
#28804
Hi cboles,

Thanks for your question!

To help you figure out where you went wrong here, please provide a detailed breakdown of how you understood the argument. Since you’re asking about a Logical Reasoning question, we expect to see evidence that you were able to:
  • Deconstruct the stimulus into premises/conclusion.
  • Understand whether the conclusion logically follows from the premises, and if not - why not?
  • Correctly identify the type of question in the stem.
  • Prephrase an answer to that question. (Don't be afraid if your prephrase was off - we still need to see what it was).
  • Defend your choice of (incorrect) answer choice.
The more you tell us about your method of approach, the better we can help you figure it out. :)

Thanks!
 Claire Horan
PowerScore Staff
  • PowerScore Staff
  • Posts: 408
  • Joined: Apr 18, 2016
|
#28809
Hi Cboles,

I am going to answer your question with some questions. First, what is the conclusion of the stimulus? It is extremely important to identify the conclusion if you want to solve a "weaken" question.

Next, if B were true, how would that affect the conclusion you identified? It may strengthen it, weaken it, or leave it unaffected.

Do the same with the other answer choices. Sometimes I write a +, -, or 0 next to each answer choice to show the effect it has on the conclusion that I have marked. Then, I do a second pass through the contenders.
 cboles
  • Posts: 27
  • Joined: Sep 15, 2016
|
#28838
I see the conclusion as the last sentence, "Adequate prenatal care, therefore, significantly decreases the risk of low birth weight babies."

That being said, this is how I evaluated the answer choices:
A) Doesn't really affect it because the babies are at normal birth weight
B) Does this weaken the conclusion because they are being marked a certain way every time the records aren't available?
C)Strengthen
D) Weaken, because it discusses how some mothers who receive adequate prental care still have low birth weight babies
E)Strengthen

Still don't understand why B is a better choice then D....
 Clay Cooper
PowerScore Staff
  • PowerScore Staff
  • Posts: 241
  • Joined: Jul 03, 2015
|
#28852
Hi cboles,

Thanks for your question!

You're correct, that is the conclusion.

Answer choice B is correct because it gives an alternate cause for the positive correlation between low birth weight and babies who are judged to have received inadequate prenatal care: a record-keeping convention (that for mothers with insufficient records and premature births, the hospitals default to marking them as having received inadequate prenatal care).

The conclusion claims that, since inadequate prenatal care is associated more strongly with low-birth-weight babies than is adequate prenatal care, it must be the cause of low-birth weight babies; but by showing how this relationship might have developed by another means - the way they keep records for premature mothers, whose babies tend to have low birth weights - we weaken that conclusion.

I hope that helps!
 cboles
  • Posts: 27
  • Joined: Sep 15, 2016
|
#28876
Got it! Thank you!
 deddiekated
  • Posts: 4
  • Joined: Apr 06, 2017
|
#34203
Not Sure if anybody still reads these, but if anybody does I'd appreciate any input on how I approached this problem:

Premise 1: Prematurely born babies are more likely to have low birth weights.
Premise 2: Mothers who received adequate prenatal care are less likely to have low birth weights.

conclusion: Adequate prenatal care decreases the risk of low birth weight babies.

I thought the hidden assumption was that Adequate Prenatal care decreases the likelihood of having prematurely born babies.

So it was rather easy to eliminate all other choices, as well as (E) which pretty much is the assumption / strengthens the argument.
Whereas, (B) weakens the argument by saying that the assumption of the argument is flawed...

Any input would be nice! Thank you!
 Emily Haney-Caron
PowerScore Staff
  • PowerScore Staff
  • Posts: 577
  • Joined: Jan 12, 2012
|
#34222
Hi deddiekated,

Thanks for the question! I wouldn't jump to the conclusion of your "hidden assumption;" be careful about drawing conclusions the stimulus doesn't give you. B weakens the argument because it tells us that babies who are often low birth weight (premature babies) may be classified as getting poor prenatal care when they really didn't, so that could be driving the relationship between prenatal care and birth weight. Does that make sense?
 Sophia123
  • Posts: 43
  • Joined: Mar 20, 2017
|
#35753
Hi,

I classified this question to be one of causal reasoning, with the cause being adequate prenatal care and the effect being decreased risk of low birth weight babies. I understand why B is the correct answer since it basically attacks the data, which is one of the ways in which we can attack causal reasoning. However, I was quite hung up by answer A since I thought it showed the effect without the cause, which I thought was another way to weaken causal reasoning. Where did I go wrong in this logic?

Thank you in advance!

-Sophia

Get the most out of your LSAT Prep Plus subscription.

Analyze and track your performance with our Testing and Analytics Package.